Least Upper Bound and the Density of the Irrationals Theorem

Click For Summary
The discussion revolves around determining the least upper bound (LUB) for the set of irrational numbers between 1 and √(7). Participants agree that √(7) is the LUB, but emphasize the need to prove it using the density of the irrationals theorem. This proof requires showing that for any ε > 0, there exists an irrational number within ε of √(7). The importance of specifying the set that x belongs to is highlighted, as it impacts the approach to solving the problem. Ultimately, the consensus is that while √(7) is indeed the LUB, justifying this requires careful consideration of the density of irrationals.
jr16
Messages
14
Reaction score
0

Homework Statement


For the following set if it has an upper bound, find two different upper bounds as well as the least upper bound (LUB), justifying your answer. If the set has no upper bound, state this and justify your answer.

{x | 1 < x < √(7) and x is irrational}
(a proof requires the density of the irrationals)

Homework Equations


density of the irrationals theorem


The Attempt at a Solution


I stated two upper bounds as √(10) and √(37)
But where I get confused is with the LUB. Isn't it simply √(7)? Why does the question state that a proof using the density of the irrationals is necessary?

Had x been declared as rational then this would be more clear to me, since there would be no LUB and I would need to prove this using the density of the rationals.

Where am I going wrong?
 
Physics news on Phys.org
hi jr

To prove that some given number is LUB, you need to prove two things. If k is the LUB First,you need to show that k is an upper bound and second, you need to prove that for any
\epsilon &gt; 0, there exists some x_1 in the set such that k-\epsilon &lt;x_1.

Can you prove these two things for \sqrt{7} ? Its here you need to use the
density of irrationals...
 
You aren't going wrong. The answer is sqrt(7) for the LUB. The answer to LUB {x | 1 < x < √(7) and x is a element of D} for any dense set D is sqrt(7). Maybe they are just asking you to prove this without proving sqrt(7) is irrational?
 
jr16 said:

Homework Statement


For the following set if it has an upper bound, find two different upper bounds as well as the least upper bound (LUB), justifying your answer. If the set has no upper bound, state this and justify your answer.

{x | 1 < x < √(7) and x is irrational}
(a proof requires the density of the irrationals)

Homework Equations


density of the irrationals theorem


The Attempt at a Solution


I stated two upper bounds as √(10) and √(37)
But where I get confused is with the LUB. Isn't it simply √(7)? Why does the question state that a proof using the density of the irrationals is necessary?

Had x been declared as rational then this would be more clear to me, since there would be no LUB and I would need to prove this using the density of the rationals.

Where am I going wrong?

I have the same puzzlement as you, so your confusion is reasonable. However, there's an important piece of information missing. When you wrote

{x | 1 < x < √(7) and x is irrational}

you did not specify the set that x is a member of. That's an important part of this problem. As you noted, if x is constrained to be rational, then we'll need to use the density of the rationals in the reals to solve this problem.

There's another reason to always include the set that x is part of when you're using set specification notation. That's because if you don't constrain x, you can get famous set theory paradoxes. So it's not only a notational formality, it's a logical necessity. That's kind of pedantic point; but in general it's helpful to get in the habit of always writing

{x in X | etc. }

And in this case, it's an important clue to the problem.
 
SteveL27 said:
I have the same puzzlement as you, so your confusion is reasonable. However, there's an important piece of information missing. When you wrote

{x | 1 < x < √(7) and x is irrational}

you did not specify the set that x is a member of. That's an important part of this problem. As you noted, if x is constrained to be rational, then we'll need to use the density of the rationals in the reals to solve this problem.

There's another reason to always include the set that x is part of when you're using set specification notation. That's because if you don't constrain x, you can get famous set theory paradoxes. So it's not only a notational formality, it's a logical necessity. That's kind of pedantic point; but in general it's helpful to get in the habit of always writing

{x in X | etc. }

And in this case, it's an important clue to the problem.

I would say {x | 1 < x < √(7) and x is irrational} is the same as saying {x is irrational | 1 < x < √(7)}. I think that is overly pedantic. I'll repeat that if you haven't shown that sqrt(7) is irrational, then the density of the irrationals may be helpful.
 
Question: A clock's minute hand has length 4 and its hour hand has length 3. What is the distance between the tips at the moment when it is increasing most rapidly?(Putnam Exam Question) Answer: Making assumption that both the hands moves at constant angular velocities, the answer is ## \sqrt{7} .## But don't you think this assumption is somewhat doubtful and wrong?

Similar threads

  • · Replies 1 ·
Replies
1
Views
2K
  • · Replies 1 ·
Replies
1
Views
1K
  • · Replies 33 ·
2
Replies
33
Views
2K
Replies
7
Views
1K
  • · Replies 3 ·
Replies
3
Views
2K
Replies
20
Views
4K
  • · Replies 8 ·
Replies
8
Views
3K
  • · Replies 3 ·
Replies
3
Views
2K
  • · Replies 10 ·
Replies
10
Views
4K
  • · Replies 4 ·
Replies
4
Views
2K